Forgot password
 Register account
View 297|Reply 2

[函数] $\mathbb{Z}^2$上的有界调和函数为常数

[Copy link]

3223

Threads

7841

Posts

52

Reputation

Show all posts

hbghlyj posted 2025-7-20 22:22 |Read mode
mathoverflow.net/q/149621
math.stackexchange.com/q/849284
math.stackexchange.com/q/51926
mathnet.ru/PresentFiles/17995/presentation.pdf
有界函数 $f:\mathbb{Z}^2\to\mathbb R$ 满足任意一点的 $f$ 值等于其相邻四点的 $f$ 值的平均值:
$$f(m,n) = \frac{f(m+1,n) + f(m-1,n) + f(m,n+1) + f(m,n-1)}{4}$$则 $f$ 为常数。
Normalization and Setup
Assume $f$ is bounded, meaning there exists $M > 0$ such that $|f(m, n)| \le M$ for all $(m, n)$. Without loss of generality, replace $f$ by $f - f(0,0)$ so that $f(0,0) = 0$.

Define the $l_1$ balls
\[
B_N = \{(m, n) \in \mathbb{Z}^2 : |m| + |n| \le N\},
\]
and set
\[
\Gamma_N = \max_{(m, n) \in B_N} f(m, n).
\]

Discrete Maximum Principle
The discrete maximum principle states that a discrete harmonic function cannot attain a strict maximum at an interior point of a domain. Thus, the maximum on $B_N$ is attained on the boundary $\partial B_N = \{(m, n) : |m| + |n| = N\}$. Since $B_N \subset B_{N+1}$, we have $\Gamma_N \le \Gamma_{N+1}$, so $\{\Gamma_N\}$ is non-decreasing. (The same holds for the minimum, but with a non-increasing sequence.)

Midpoint-Convexity and Growth Estimate
Consider a point $(m, n) \in \partial B_N$. The neighbors fall into $B_{N-1}$ or $B_{N+1}$.
  • For "interior" boundary points (not vertices), two neighbors are in $B_{N-1}$ and two in $B_{N+1}$, so
    \[
    f(m, n) \le \frac{2\Gamma_{N+1} + 2\Gamma_{N-1}}{4} = \frac{\Gamma_{N+1} + \Gamma_{N-1}}{2}.
    \]
  • For vertex points (e.g., $(N, 0)$), three neighbors are in $B_{N+1}$ and one in $B_{N-1}$, so
    \[
    f(m, n) \le \frac{3\Gamma_{N+1} + \Gamma_{N-1}}{4}.
    \]
Since $\Gamma_{N+1} \ge \Gamma_{N-1}$ and the maximum may be at a vertex, the controlling bound is the weaker one:
\[
\Gamma_N \le \frac{3\Gamma_{N+1} + \Gamma_{N-1}}{4}.
\]
Rearranging gives
\[
\Gamma_{N+1} - \Gamma_N \ge \frac{1}{3} (\Gamma_N - \Gamma_{N-1}).
\]
Let $d_N = \Gamma_N - \Gamma_{N-1} \ge 0$ (with $\Gamma_0 = 0$, so $d_1 = \Gamma_1$). Then $d_{N+1} \ge \frac{1}{3} d_N$, and by induction,
\[
d_{N+1} \ge \frac{\Gamma_1}{3^N}.
\]
Thus,
\[
\Gamma_N = \sum_{k=1}^N d_k \ge \sum_{k=1}^N \frac{\Gamma_1}{3^{k-1}} = \Gamma_1 \frac{1 - (1/3)^N}{1 - 1/3} = \frac{3}{2} \Gamma_1 \left(1 - \frac{1}{3^N}\right).
\]
If $\Gamma_1 > 0$, then $\Gamma_N \ge \frac{3}{2} \Gamma_1 (1 - 1/3^N)$, approaching $\frac{3}{2} \Gamma_1$ as $N \to \infty$.

Conclusion
Assume $\Gamma_1 > 0$ (otherwise, by monotonicity, $\Gamma_N = 0$ for all $N$, so $f \le 0$; applying to $-f$ shows $f = 0$). The lower bound suggests $\Gamma_N$ approaches a finite limit, but this assumes equality can be achieved at each step. However, achieving equality requires the maximum on $\partial B_N$ to be at a vertex with neighbors attaining the exact maxima, leading to inconsistencies in the harmonic condition at non-vertex points where the maximum is also forced.

For example, assuming equality propagates the maximum to certain points, but at non-vertex boundary points (where the neighbor distribution is 2-2), the harmonic average cannot match the required value, as it is bounded above by $(\Gamma_{N+1} + \Gamma_{N-1})/2 < \Gamma_N$ under the equality assumptions. This contradiction implies the actual growth must exceed the lower bound, forcing $\Gamma_N \to \infty$, contradicting boundedness.

Thus, $\Gamma_1 = 0$, so $f \equiv 0$ (shifting back, $f$ is constant).

48

Threads

770

Posts

93

Reputation

Show all posts

Czhang271828 posted 2024-10-6 12:26
原来的问题, 考虑 $\sup f$ 即可.

实际上, $f$ 上有界 (或者下有界) 就行. 考虑图上的随机过程. 记一族 $t$-参数的随机变量 $\{X_t\}_{t\in \mathbb N}$. 由于

  • 下一次观测值的条件期望等于本次观测值 ($\mathbb E[f(X_{n+1})\mid f(X_1),\ldots, f(X_n)]=X_n$),
  • 所有 $f(X_t)$ 期望有限.

从而随机过程 $f(X_t)$ 是离散时间鞅. 依照著名的离散时间鞅的收敛定理, $f(X_t)\to f(X_\infty)$ a.e..

由于 $X_t$ 在每一点处是 infinitely recurrent 的, 从而 $f(X_t)$ 在除去有限个点之外是常数, 因此只能是常数.

3223

Threads

7841

Posts

52

Reputation

Show all posts

original poster hbghlyj posted 2025-7-20 23:00
对证明的最后一步的疑问已发到 math.stackexchange.com/q/5084301 求助

Quick Reply

Advanced Mode
B Color Image Link Quote Code Smilies
You have to log in before you can reply Login | Register account

$\LaTeX$ formula tutorial

Mobile version

2025-7-22 22:06 GMT+8

Powered by Discuz!

Processed in 0.011872 seconds, 22 queries